Math, asked by vishagh, 1 year ago

If tan 35° = k then find the value of
(tan 145° - tan 125°)/(1+tan145°.tan125°)

Answers

Answered by MaheswariS
19

\textbf{Given:}

\mathsf{tan\,35^\circ=k}

\textbf{To find:}

\textsf{The value of}\;\mathsf{\dfrac{tan145^\circ-tan125^\circ}{1+tan145^\circ\;tan125^\circ}}

\textbf{Solution:}

\mathsf{Consider,}

\mathsf{\dfrac{tan145^\circ-tan125^\circ}{1+tan145^\circ\;tan125^\circ}}

\mathsf{=\dfrac{tan(180^\circ-35^\circ)-tan(90^\circ+35^\circ)}{1+tan(180^\circ-35^\circ)\;tan(90^\circ+35^\circ)}}

\mathsf{=\dfrac{-tan35^\circ+cot35^\circ}{1+tan35^\circ\;cot35^\circ}}

\mathsf{=\dfrac{-tan35^\circ+\dfrac{1}{tan35^\circ}}{1+tan35^\circ(\dfrac{1}{tan35^\circ})}}

\mathsf{=\dfrac{-k+\dfrac{1}{k}}{1+1}}

\mathsf{=\dfrac{\dfrac{1-k^2}{k}}{2}}

\mathsf{=\dfrac{1-k^2}{2k}}

\implies\boxed{\mathsf{\dfrac{tan145^\circ-tan125^\circ}{1+tan145^\circ\;tan125^\circ}=\dfrac{1-k^2}{2k}}}

\textbf{Find more:}

If 3x=cosecθ and 3/x=cotθ, then find the value of 3(x²-1/x²) PLEASE HELP BEST ANSWER WILL BE MARKED BRAINLIEST

https://brainly.in/question/21134289

If sinθ+cosθ=x prove that sin6θ+cos6θ=4-3(x²-1)²/4

https://brainly.in/question/15923140

Answered by sakshamsingh80009
0

Answer:

an35

=k

\textbf{To find:}To find:

\textsf{The value of}\;\mathsf{\dfrac{tan145^\circ-tan125^\circ}{1+tan145^\circ\;tan125^\circ}}The value of

1+tan145

tan125

tan145

−tan125

\textbf{Solution:}Solution:

\mathsf{Consider,}Consider,

\mathsf{\dfrac{tan145^\circ-tan125^\circ}{1+tan145^\circ\;tan125^\circ}}

1+tan145

tan125

tan145

−tan125

\mathsf{=\dfrac{tan(180^\circ-35^\circ)-tan(90^\circ+35^\circ)}{1+tan(180^\circ-35^\circ)\;tan(90^\circ+35^\circ)}}=

1+tan(180

−35

)tan(90

+35

)

tan(180

−35

)−tan(90

+35

)

\mathsf{=\dfrac{-tan35^\circ+cot35^\circ}{1+tan35^\circ\;cot35^\circ}}=

1+tan35

cot35

−tan35

+cot35

\mathsf{=\dfrac{-tan35^\circ+\dfrac{1}{tan35^\circ}}{1+tan35^\circ(\dfrac{1}{tan35^\circ})}}=

1+tan35

(

tan35

1

)

−tan35

+

tan35

1

\mathsf{=\dfrac{-k+\dfrac{1}{k}}{1+1}}=

1+1

−k+

k

1

\mathsf{=\dfrac{\dfrac{1-k^2}{k}}{2}}=

2

k

1−k

2

\mathsf{=\dfrac{1-k^2}{2k}}=

2k

1−k

2

\implies\boxed{\mathsf{\dfrac{tan145^\circ-tan125^\circ}{1+tan145^\circ\;tan125^\circ}=\dfrac{1-k^2}{2k}}}⟹

1+tan145

tan125

tan145

−tan125

=

2k

1−k

2

Similar questions